Which one of the following, if true, would most call into question the author's assertion in the last sentence of the...

Nishant-Varma on September 28, 2021

Why not B?

Does that not weaken the claim as well?

Replies
Create a free account to read and take part in forum discussions.

Already have an account? log in

jingjingxiao11111@gmail.com on December 25, 2021

Hi I am not an instructor but I will try to help. I also got this question wrong by the way.

The last sentence reads: “Because the Danish agency, . unlike its U.S. counterpart, recognized the importance (60) of local involvement at all levels, the project has a . good chance of remaining competitive and profitable . for the long run.”

As you can see above, the last sentence is forecasting that the Indian project, in collaboration with the Danish agency, will remain profitable in the long run due to their recognition of the importance of local involvement at all levels.

To weaken this, we need an answer choice that calls into question the reason behind why the Indian project in collaboration with Danish agency will remain profitable.

A) gives this alternative reason. A) states “The profitability of the India project was due primarily to temporary subsidies from the Indian government.”

According to A), the reason behind Indian project’s profitability was due not to local involvement. Rather, the project’s profitability was due to money from the Indian government. Because A) gives an alternative reason as to why the Indian project is profitable, A) weakens the argument made in the last sentence.


Now let’s take a look at B). B) reads: “The Danish energy agency invested more funds in the India project than the U.S. agency invested in the Brazil project.”

I believe B) is irrelevant to the last sentence of the passage. B) does not give an alternative reason as to why the Indian project (collaborated with Danish) is especially profitable. Unlike A), B) makes no mention of the reason behind Indian project’s profitability. Thus B) has no bearing on the argument made in the last sentence.

I hope that it helps. Please feel free to correct me. Thank you.

Emil-Kunkin on January 27, 2022

Hi all- just wanted to chime in that @jingjingxaio1111 is completely correct! A does indeed weaken by providing a different reason for profitability, and B doesn't have any real bearing on the final sentence.

jingjingxiao11111@gmail.com on February 11, 2022

Thank you! It is always good to get feedbacks from LSATMax instructors that I am on the right track!

Ravi on February 13, 2022

Happy it helped!

Ravi on February 13, 2022

Happy it helped!